Mathcenter Forum  

Go Back   Mathcenter Forum > ค้นหาในห้อง
สมัครสมาชิก คู่มือการใช้ รายชื่อสมาชิก ปฏิทิน ข้อความวันนี้

แสดงผลลัพธ์ตั้งแต่ 1 ถึง 2 จากทั้งหมด 2
ใช้เวลาค้นหา 0.00 วินาที.
ค้นหา: ข้อความของคุณ: S!xTo12Y
ห้อง: อสมการ 09 พฤษภาคม 2009, 23:52
คำตอบ: 5
เปิดอ่าน: 2,852
ข้อความของคุณ S!xTo12Y
ขอขอบคุณคุณ RoSe-JoKer...

ขอขอบคุณคุณ RoSe-JoKer มากนะครับสำหรับโจทย์ที่มีความสวยมากมาย

ได้ว่าอสมการเป็นจริง
$\Leftrightarrow \displaystyle{\sum_{cyc}}\frac{a}{b+c}-\frac{3}{2} \geqslant...
ห้อง: อสมการ 09 พฤษภาคม 2009, 22:31
คำตอบ: 10
เปิดอ่าน: 3,272
ข้อความของคุณ S!xTo12Y
จากโจทย์ โดยอสมการโคชีได้ว่า $L.H.S =...

จากโจทย์ โดยอสมการโคชีได้ว่า $L.H.S = \sum_{cyc}\frac{a^4}{a^3+a^2b+ab^2}\geqslant \frac{(a^2+b^2+c^2)^2}{a^3+b^3+c^3+a^2b+ab^2+b^2c+bc^2+c^2a+ca^2} =\frac{a^2+b^2+c^2}{a+b+c}$
ซึ่งโดยอสมการ Power...
แสดงผลลัพธ์ตั้งแต่ 1 ถึง 2 จากทั้งหมด 2

 
ทางลัดสู่ห้อง

เวลาที่แสดงทั้งหมด เป็นเวลาที่ประเทศไทย (GMT +7) ขณะนี้เป็นเวลา 00:23


Powered by vBulletin® Copyright ©2000 - 2024, Jelsoft Enterprises Ltd.
Modified by Jetsada Karnpracha